Đến nội dung

HoangKhanh2002

HoangKhanh2002

Đăng ký: 12-07-2016
Offline Đăng nhập: 19-03-2022 - 15:33
***--

#690858 $\left\{\begin{matrix} y^{3}+\sqrt{8x^{4}-2y}=2(2x^{...

Gửi bởi HoangKhanh2002 trong 18-08-2017 - 11:51

Giải hệ phương trình:

 

 $\left\{\begin{matrix} y^{3}+\sqrt{8x^{4}-2y}=2(2x^{4}+3) & & \\ \sqrt{2x^{2}+x+y}+2\sqrt{x+2y}=\sqrt{9x-2x^{2}+19y} & & \end{matrix}\right.$

 

Mọi người giúp em với ạ. Em cần gấp ạ

Dùng liên hợp

ĐK: $\left\{\begin{matrix} 8x^4-2y\geqslant 0\\ 2x^2+x+y\geqslant 0\\ x+2y\geqslant 0\\ 9x-2x^2+19y\geqslant 0 \end{matrix}\right.$

Từ phương trình thứ hai ta có:

$\sqrt{2x^{2}+x+y}+2\sqrt{x+2y}=\sqrt{9x-2x^{2}+19y}\\ \iff \sqrt{2x^2+x+y}-\sqrt{x+2y}=\sqrt{9x-2x^2+19y}-3\sqrt{x+2y}\\\iff \dfrac{2x^2-y}{\sqrt{2x^2+x+y}+\sqrt{x+2y}}=\dfrac{y-2x^2}{\sqrt{9x-2x^2+19y}+3\sqrt{x+2y}}\\\iff (2x^2-y)\left ( \dfrac{1}{\sqrt{2x^2+x+y}+\sqrt{x+2y}}+\dfrac{1}{\sqrt{9x-2x^2+19y}+3\sqrt{x+2y}} \right )=0\\ \iff y=2x^2$

Thay vào phương trình đầu ta có:

$y^3+\sqrt{2y^2-2y}=y^2+6\iff y^3-y^2+y-6+\sqrt{2y^2-2y}-y=0\\\iff (y-2)(y^2+y+3)+\dfrac{y(y-2)}{\sqrt{2y^2-2y}+y}=0\\\iff (y-2)\left ( y^2+y+3+\dfrac{y}{\sqrt{2y^2-2y}+y} \right )=0$

Nhận thấy:

$y^2+y+3+\dfrac{y}{\sqrt{2y^2-2y}+y}>y^2+y+3+\dfrac{2y}{2y^2+1}\\=y^2+y+1+\dfrac{4y^2+2y+2}{2y^2+1}>0$

Vậy hệ có nghiệm: $\boxed{(x;y)\in \left \{ (-1;2);(1;2) \right \}}\hspace{1cm}\square$




#690848 $\sum \frac{1}{\sqrt{8a+1}}...

Gửi bởi HoangKhanh2002 trong 18-08-2017 - 10:46

Cho $abc=1$ CMR : $\sum \frac{1}{\sqrt{8a+1}}\geq 1$

Mình làm theo phản chứng như sau:

Đặt: $x=\dfrac{1}{\sqrt{8a+1}};y=\dfrac{1}{\sqrt{8b+1}};z=\dfrac{1}{\sqrt{8c+1}}\Rightarrow x,y,z>0$

Từ đó, rút: $a=\dfrac{1-x^2}{8x^2};b=\dfrac{1-y^2}{8y^2};a=\dfrac{1-z^2}{8z^2}\\\Rightarrow 8^3x^2y^2z^2=(1-x^2)(1-y^2)(1-z^2)$

Ta cần chứng minh: $x+y+z \geqslant 1$

Giả sử ngược lại: $x+y+z < 1$

Khi đó: $1-x^2>(x+y+z)^2-x^2=(y+z)((x+y)+(z+x))\\\geqslant^{AM-GM} 2(y+z)\sqrt{(x+y)(y+z)}>0$

Tương tự: $1-y^2\geqslant^{AM-GM}2(x+z)\sqrt{(x+y)(y+z)}>0\\ 1-z^2\geqslant^{AM-GM}2(x+y)\sqrt{(x+z)(y+z)}>0$

Nhân vế với vế 3 BĐT trên:

$8^3x^2y^2z^2=(1-x^2)(1-y^2)(1-z^2)>8(x+y)^2(y+z)^2(z+x)^2\\\Rightarrow 8xyz>(x+y)(y+z)(z+x)$

BĐT cuối mâu thuẫn

Vậy ta có điều phải chứng minh




#690784 [CHUYÊN ĐỀ] CHỨNG MINH BẤT ĐẲNG THỨC

Gửi bởi HoangKhanh2002 trong 17-08-2017 - 20:03

       Cho các số thực dương a, b,c. Chứng minh rằng :

               $\frac{2(a^{3}+b^{3}+c^{3})}{abc}+\frac{9(a+b+c)^{2}}{a^{2}+b^{2}+c^{2}}\geqslant 33$

Dùng S.O.S như sau:

BĐT cần chứng minh tương đương với:

$\dfrac{2(a^3+b^3+c^3)}{abc}-6+\dfrac{9(a+b+c)^2}{a^2+b^2+c^2}-27\geqslant 0\\ \iff \dfrac{2(a+b+c)(a^2+b^2+c^2-ab-bc-ca)}{abc}+\dfrac{18(ab+bc+ca-a^2-b^2-c^2)}{a^2+b^2+c^2}\geqslant 0\\\iff \left [ (a-b)^2+(b-c)^2+(c-a)^2 \right ]\left ( \dfrac{a+b+c}{abc}-\dfrac{9}{a^2+b^2+c^2} \right )\geqslant 0$

Mà: $\dfrac{a+b+c}{abc}-\dfrac{9}{a^2+b^2+c^2}=\dfrac{1}{ab}+\dfrac{1}{bc}+\dfrac{1}{ca}-\dfrac{9}{a^2+b^2+c^2}\\\geqslant \dfrac{9}{ab+bc+ca}-\dfrac{9}{a^2+b^2+c^2}\geqslant 0$

Do đó: $\implies Q.E.D$




#690778 tìm x

Gửi bởi HoangKhanh2002 trong 17-08-2017 - 19:35

ai giải được bài này em mới khâm phục. mọi người giải giúp em với được ko ạ

2017-07-30.png

Bài này làm như sau:

ĐK: $x \geqslant -\dfrac{3}{4}$

Phương trình tương đương:

$x^3-x^2-x+1=\sqrt{4x+3}+\sqrt{3x^2+10x+6}\\ \iff x(x^2-2x-2)+(x+1-\sqrt{4x+3})+(x^2-\sqrt{3x^2+10x+6})=0\\ \iff x(x^2-2x-2)+\dfrac{x^2-2x-2}{x+1+\sqrt{4x+3}}+\dfrac{(x^2-2x-2)(x^2+2x+3)}{x^2+\sqrt{3x^2+10x+6}}=0\\ \iff (x^2-2x-2)\left ( \dfrac{x^2+x+x\sqrt{4x+3}+1} {x+1+\sqrt{4x+3}}+\dfrac{x^2+2x+3}{x^2+\sqrt{3x^2+10x+6}}\right )=0$

Dễ thấy biểu thức trong ngoặc dương

$\implies x^2-2x-2=0\iff x=1\pm \sqrt{3}(TM)$

Vậy phương trình đã cho có nghiệm: $x=1\pm \sqrt{3}\hspace{0.5cm}\square$

PS: Lâu mới quay lại diễn đàn, không biết còn ai nhớ tui không!!!!




#687032 cho $x^{2}+y^{2}+z^{2} =3$. C/m:...

Gửi bởi HoangKhanh2002 trong 09-07-2017 - 10:54

cho $x^{2}+y^{2}+z^{2} =3$. C/m: $\frac{x}{3-yz}+\frac{y}{3-xz}+\frac{z}{3-xy} \leqslant \frac{3}{2}$

Ta có:

$\sum \dfrac{x}{3-yz}\leqslant \sum \dfrac{x}{3-\dfrac{y^2+z^2}{2}}=\sum \dfrac{2x}{3+x^2}$

BĐT phụ: $\dfrac{2a}{3+a^2}\leqslant \dfrac{1}{8}a^2+\dfrac{3}{8}\iff \dfrac{(a-1)^2((a+1)^2+8)}{8(3+a^2)}\geqslant 0$ (luôn đúng)

Do đó: $\sum \dfrac{x}{3-yz}\leqslant\sum \dfrac{2x}{3+x^2}\leqslant \dfrac{3}{2}$

Dấu "=" xảy ra: $\iff x=y=z=1 \hspace{0,5cm} \square$




#686870 Đề luyện tập olympic khối 10 VMF lần 1 tháng 7

Gửi bởi HoangKhanh2002 trong 07-07-2017 - 21:46

$\boxed{\text{Bài 3}}$

      b) Tam giác $ABC$ có các cạnh $a, b, c$ và bán kính đường tròn ngoại tiếp, nội

tiếp lần lượt là $R, r$ thỏa mãn đẳng thức:               .

                                      $\dfrac{a^3+b^3+c^3}{abc}+\dfrac{2r}{R}=4$

Chứng minh tam giác $ABC$ đều

$\boxed{\text{Lời giải bài 3b}}$

Ta có: $S_{ABC}=p.r=\dfrac{abc}{4R}=\sqrt{p(p-a)(p-b)(p-c)}\implies \dfrac{2r}{R}=\dfrac{8S_{ABC}^2}{pabc}$

Từ giả thiết: $\dfrac{a^3+b^3+c^3}{abc}+\dfrac{2r}{R}=4 \iff \dfrac{a^3+b^3+c^3}{abc}+\dfrac{8S_{ABC}^2}{pabc}=4\\ \iff p(a^3+b^3+c^3)+8.p.\dfrac{a+b-c}{2}.\dfrac{a-b+c}{2}.\dfrac{-a+b+c}{2}=4abcp\\ \iff a^3+b^3+c^3+(a+b-c)(a-b+c)(-a+b+c)-4abc=0\\ \iff a^2b+a^2c+ab^2-6abc+ac^2+b^2c+bc^2=0\\ \iff b(a-c)^2+c(a-b)^2+a(b-c)^2=0\\ \iff a=b=c \hspace{0,5cm}\square$

PS: Các bạn nhớ đọc kĩ quy định khi post. Trích lại đề bài đó và sử dụng lệnh \boxed{\text{Lời giải bài..}}




#686863 Đề luyện tập olympic khối 10 VMF lần 1 tháng 7

Gửi bởi HoangKhanh2002 trong 07-07-2017 - 21:08

Khởi động nào các mem

$\boxed{\text{ĐỀ SỐ 1}}$ (Dành cho HSG THPT không chuyên)

$\boxed{\text{Bài 1}}$

a) Giải hệ phương trình: $\left\{\begin{matrix} \sqrt{5x^2+2xy+2y^2}+\sqrt{2x^2+2xy+5y^2}=3(x+y)\\ \sqrt{2x+y+1}+2\sqrt[3]{7x+12y+8}=2xy+y+5 \end{matrix}\right.$

b) Gọi $x_{1},x_{2}$ là hai nghiệm của tam thức: $f(x)=x^2+ax+b$ với $a,b \in[-1,1]$. Chứng minh: $(\left | x_{1} \right |+1)(\left | x_{2} \right |+1) \leqslant 2+\sqrt{5}$

$\boxed{\text{Bài 2}}$ Tìm tất cả các giá trị của $m$ để bất phương trình sau có nghiệm:

$\sqrt{x-1}-2\sqrt[4]{x^2-x}+m\sqrt{x}\leqslant 0$

$\boxed{\text{Bài 3}}$

      a) Cho tam giác $ABC$ nội tiếp đường tròn $(O)$. Gọi $I$ là trung điểm của $AC$ và $M$ là điểm thoả mãn $\overrightarrow{OM}=2\overrightarrow{OA}+\overrightarrow{OB}+\overrightarrow{OC}$. Biết rằng $OM$ vuông góc với $BI$ và $AC^2=3BC.BA$. Tính $\widehat{ABC}$

      b) Tam giác $ABC$ có các cạnh $a, b, c$ và bán kính đường tròn ngoại tiếp, nội

tiếp lần lượt là $R, r$ thỏa mãn đẳng thức:               .

                                      $\dfrac{a^3+b^3+c^3}{abc}+\dfrac{2r}{R}=4$

Chứng minh tam giác $ABC$ đều

$\boxed{\text{Bài 4}}$

       Trong mặt phẳng với hệ tọa độ $Oxy$, cho tam giác $ABC$. Gọi $H,K$ lần lượt là chân đường cao hạ từ các đỉnh $B,C$ của tam giác $ABC$. Tìm tọa độ các đỉnh của tam giác $ABC$ biết $H(5,-1),K\left ( \dfrac{1}{5},\dfrac{3}{5} \right )$, phương trình đường thẳng $BC$ là $x+3y+4=0$ và điểm $B$ có hoành độ âm.

$\boxed{\text{Bài 5}}$

     Cho $a,b,c$ dương. Chứng minh rằng:

$a^2+b^2+c^2+a^2b^2c^2\geq 2(ab+bc+ca-1)$

File pdf (in ấn): File gửi kèm  DE_LAN_1.pdf   120.5K   294 Số lần tải ($\LaTeX{}$ bởi: Nguyễn Phúc Tăng)

Spoiler




#686833 Đề luyện tập olympic khối 10 VMF lần 1 tháng 7

Gửi bởi HoangKhanh2002 trong 07-07-2017 - 17:30

    Kì thi HSG tỉnh, Olympic 30/4 là một kì thi đầy cam go, quyết liệt đối với các bạn học sinh, nhất là các bạn học sinh trường chuyên.  Nói như vậy để thấy rằng, vượt qua kì thi và dành 1 kết quả cao là một thử thách lớn đối với các thí sinh, và điều đó đòi hỏi sự chuẩn bị, ôn tập kĩ lưỡng và những kĩ năng vững vàng đến từ các bạn. Nhằm đáp ứng nhu cầu bức thiết về mặt kiến thức đó, thiết nghĩ cần có 1 topic cần được lập ra để cho chúng ta rèn luyện và củng cố. Em xin thay mặt cho các thành viên 2k2 trên diễn đàn lập ra topic này.

     Topic xin đưa ra những quy định như sau:

- Tuyệt đối không spam. Trước mỗi bài sử dụng lệnh \boxed{\text{Bài....}}, trước lời giải sử dụng lệnh \boxed{\text{Lời giải bài....}}

- Post đề nào, giải xong đề đó. Tránh đăng tràn lan gây loãng topic

- Giải bài nào chỉ trích dẫn bài đó, không trích dẫn toàn văn gây xấu topic.

     Mỗi tuần Topic sẽ đăng đúng 1 đề có cấu trúc giống 1 đề chính thức, và sẽ thảo luận đúng trong tuần đó để giải quyết hết các bài tập trong đề đó.

Nếu các bạn có mong muốn đóng góp cho Topic hãy nhắn tin trực tiếp với bạn HoangKhanh2002 để gửi đề.

       Rất mong các bạn gần xa sẽ ủng hộ thế mạnh của mình để các mem ở Topic ta tham gia thi đạt KQ tốt nhất. Mong các ĐHV THPT, ĐHV OLYMPIC tham gia nhiệt tình

Xin chân thành cảm ơn anh Lê Hoàng Bảo (Baoriven) đã giúp đỡ!!!




#686746 Max P=$\frac{1}{\sqrt{a+3b}}+...

Gửi bởi HoangKhanh2002 trong 06-07-2017 - 23:07

a,b,c là các số thực dương thỏa mãn $\frac{1}{\sqrt{a}}+\frac{1}{\sqrt{b}}+\frac{1}{\sqrt{c}}=2$ 

Tìm Max P=$\frac{1}{\sqrt{a+3b}}+\frac{1}{\sqrt{b+3c}}+\frac{1}{\sqrt{c+3a}}$

Áp dụng $AM-GM$ ta có:

$P=\dfrac{1}{\sqrt{a+3b}}+\dfrac{1}{\sqrt{b+3c}}+\dfrac{1}{\sqrt{c+3a}}\leqslant \dfrac{1}{\sqrt{4\sqrt[4]{ab^3}}}+ \dfrac{1}{\sqrt{4\sqrt[4]{bc^3}}}+ \dfrac{1}{\sqrt{4\sqrt[4]{ca^3}}}\\ =\dfrac{1}{2}\left ( \dfrac{1}{\sqrt[8]{ab^3}}+ \dfrac{1}{\sqrt[8]{bc^3}}+ \dfrac{1}{\sqrt[8]{ca^3}}\right )\leqslant \dfrac{1}{8}\left ( \dfrac{1}{\sqrt{a}}+\dfrac{3}{\sqrt{b}}+\dfrac{1}{\sqrt{b}}+\dfrac{3}{\sqrt{c}}+\dfrac{1}{\sqrt{c}}+\dfrac{3}{\sqrt{a}} \right ) \\=\dfrac{1}{2}\left ( \dfrac{1}{\sqrt{a}} +\dfrac{1}{\sqrt{b}} +\dfrac{1}{\sqrt{c}} \right )=1$

Dấu "=" xảy ra: $\iff a=b=c=\dfrac{9}{4} \hspace{0,5cm} \square$




#686531 $\left\{\begin{matrix}2x^3-4x^2+3x-1=2x^3(2-y)\sqrt{...

Gửi bởi HoangKhanh2002 trong 05-07-2017 - 10:34

Giải hệ phương trình:

$\left\{\begin{matrix}(17-3x)\sqrt{5-x}+(3y-14)\sqrt{4-y}=0 & & \\ 2\sqrt{2x+y+5}+3\sqrt{3x+2y+11}=x^2+6x+13 & & \end{matrix}\right.$

ĐK: $\left\{\begin{matrix} x\leqslant 5\\ y\leqslant 4\\ 2x+y+5\geqslant 0\\ 3x+2y+11\geqslant 0 \end{matrix}\right.$

Phương trình thứ nhất viết lại dưới dạng: $(2+3(5-x))\sqrt{5-x}=(2+3(4-y))\sqrt{4-y}$

Xét hàm $f(t)=(2+3t^2)t\implies f'(t)=9t^2+2>0\implies f(t)$ đồng biến trên $\mathbb{R}$ $\implies 5-x=4-y \iff x-y=1$

Thay vào phương trình thứ 2:

$\iff 2\sqrt{3x+4}+3\sqrt{5x+9}=x^2+6x+13$

Đến đây tư giải




#686491 ĐỀ THI THPT CHUYÊN TRẦN PHÚ HẢI PHÒNG.

Gửi bởi HoangKhanh2002 trong 04-07-2017 - 19:12

Câu hệ: Chuyển về đồng bậc thôi

$\left\{\begin{matrix} x^3+xy^2-10y=0\\ x^2+6y^2=10 \end{matrix}\right. \iff x^3+xy^2-y(x^2+6y^2)=0 \\\iff (x-2y)(x^2+xy+3y^2)=0$

Câu bất:

Sử dụng BĐT $Cauchy-Schwarz$ dạng $Engel$ ta có:

$\sum_{cyc} \dfrac{a^4}{b^3(a+2c)}=\sum_{cyc}\dfrac{\dfrac{a^4}{b^2}}{b(a+2c)}\geqslant \sum_{cyc}\dfrac{\left (\sum \dfrac{a^2}{b} \right )^2}{3(ab+bc+ca)}\geqslant \dfrac{(a+b+c)^2}{(a+b+c)^2}=1$

Dấu "=" xảy ra: $\iff a=b=c \hspace{0,5cm} \square$




#685271 CMR: $\sum \frac{b\sqrt{c}}{a(...

Gửi bởi HoangKhanh2002 trong 21-06-2017 - 15:41

Cho $a,b,c> 0$ và thỏa mãn: $a+b+c=1$. Chứng minh rằng:

$\frac{b\sqrt{c}}{a(\sqrt{3c}+\sqrt{ab})}+\frac{c\sqrt{a}}{b(\sqrt{3a}+\sqrt{bc})}+\frac{a\sqrt{b}}{c(\sqrt{3b}+\sqrt{ca})}\geq \frac{3\sqrt{3}}{4}$

Biến đổi vế trái: $\dfrac{b\sqrt{c}}{a(\sqrt{3c}+\sqrt{ab})}+\dfrac{c\sqrt{a}}{b(\sqrt{3a}+\sqrt{bc})}+\dfrac{a\sqrt{b}}{c(\sqrt{3b}+\sqrt{ca})}=\dfrac{\sqrt{\dfrac{bc}{a}}}{3\sqrt{\dfrac{3ca}{b}}+a}+\dfrac{\sqrt{\dfrac{ac}{b}}}{3\sqrt{\dfrac{3ab}{c}}+b}+\dfrac{\sqrt{\dfrac{ab}{c}}}{3\sqrt{\dfrac{3bc}{a}}+c}$

Đặt: $\sqrt{\dfrac{bc}{a}}=x,\sqrt{\dfrac{ac}{b}}=y,\sqrt{\dfrac{bc}{a}}=z \implies xy+yz+zx=1$

Áp dụng BĐT $Cauchy-Schwarz$ dạng $Engel$ ta có:

$\sum \dfrac{x}{\sqrt{3}y+yz}=\sum \dfrac{x^2}{\sqrt{3}xy+xyz}\geqslant \dfrac{(x+y+z)^2}{\sqrt{3}+xyz}$

Mà: $(x+y+z)^2\geqslant 3(xy+yz+zx)=3, xy+yz+zx\geqslant 3\sqrt[3]{x^2y^2z^2}$

Do đó: $\sum \dfrac{x}{\sqrt{3}y+yz}\geqslant \dfrac{3}{\sqrt{3}+\dfrac{1}{\sqrt{3}}}=\dfrac{3\sqrt{3}}{4}$

Dấu "=" xảy ra: $\iff a=b=c=\dfrac{1}{3} \hspace{0.5cm} \square$




#685133 CMR: $(a+b-c)^2(b+c-a)^2(c+a-b)^2 \geq (a^2+b^2-c^2)(b^2+c^2-a^2)(c...

Gửi bởi HoangKhanh2002 trong 20-06-2017 - 12:25

Cho a,b,c là các số thực tùy ý

CMR: $(a+b-c)^2(b+c-a)^2(c+a-b)^2 \geq (a^2+b^2-c^2)(b^2+c^2-a^2)(c^2+a^2-b^2)$

Nhận thấy, nếu $a^2, b^2, c^2$ không là độ dài ba cạnh của tam giác thì trong 3 số $a^{2}+b^{2}-c^{2}; b^{2}+c^{2}-a^{2}; c^{2}+a^{2}-b^{2}$ sẽ có 2 số $\geqslant 0$, 1 số $\leqslant 0$ hoặc cả 3 số $\leqslant 0$ nên BĐT luôn đúng vì $VT>0>VP$

Xét trường hợp các số trên là 3 cạnh của tam giác

BĐT đã cho tương đương với:

$(a^2-(b-c)^2)(b^2-(c-a)^2)(c^2-(a-b)^2)\geqslant (a^{2}+b^{2}-c^{2})(b^{2}+c^{2}-a^{2})(c^{2}+a^{2}-b^{2})$

Ta chứng minh: $(a^2-(b-c)^2)^2\geqslant (a^2-b^2+c^2)(a^2+b^2-c^2)$

Thật vậy, bất đẳng thức đã cho tương đương với: $(b-c)^2(b^2+c^2-a^2)$ (đúng)

Lập các biểu thức hoán vị, nhân theo vế có ngay $Q.E.D$.




#684971 Cho $ABC$ có phân giác $AD$

Gửi bởi HoangKhanh2002 trong 19-06-2017 - 09:11

AD.png

 

Giải như sau:

Kẻ $AD$ cắt $(ABC)$ tại $E$

Gọi $F$ là tâm $(ABC)$

Ta có: $\dfrac{1}{BD^2}+\dfrac{1}{CD^2}=\dfrac{2}{AD^2} \iff \dfrac{2}{AD^2}=\dfrac{BD^2+CD^2}{BD^2.CD^2}=\dfrac{BD^2+CD^2}{AD^2DE^2}\\\iff 2DE^2=BD^2+CD^2=(BF-FD)^2+(BF+FD)^2=2(BF^2+DF^2)\\ \iff DE^2-DF^2=BF^2 \iff BF^2=EF^2$ (điều này hiển nhiên)




#684948 $\left\{\begin{matrix} x+\frac{3...

Gửi bởi HoangKhanh2002 trong 18-06-2017 - 21:40

Giải HPT: $\left\{\begin{matrix} x+\frac{3x-y}{x^2+y^2}=3\\y-\frac{x+3y}{x^2+y^2}=0 \end{matrix}\right.$

ĐK: $x,y$ không cùng bằng 0

Xét $x=0$ không tìm được $y$

Xét $y=0\implies$ không có x

Xét $x\neq 0,y\neq 0$. Hệ đã cho tương đương với: $\left\{\begin{matrix} xy+\dfrac{xy+3y^2}{x^2+y^2}=3y\\ xy-\dfrac{xy-3x^2}{x^2+y^2}=0 \end{matrix}\right.\implies 2xy+3=3y\implies x=\dfrac{3y-3}{2}$

Thay vào PT(2) và giải thôi $\iff \dfrac{(3y-3)^2}{4y}+y^3-y+\dfrac{9y-9}{2y}=0 \iff 4y^4+5y^2-9=0$

Vậy: $\boxed{(x,y)\in \left \{ (1,-1);(2,1) \right \}}$